+0  
 
+5
539
4
avatar+21 

 

(n is a natural number)

 Oct 22, 2015
edited by JakeTheWiseElf  Oct 22, 2015
edited by JakeTheWiseElf  Oct 22, 2015
edited by JakeTheWiseElf  Oct 22, 2015
edited by JakeTheWiseElf  Oct 22, 2015
edited by JakeTheWiseElf  Oct 22, 2015

Best Answer 

 #2
avatar+118654 
+10

 

Hello once again Jake   laugh

 

Prove by induction     (n is  a nattural number)

 

\(1+\frac{1}{4}+\frac{1}{9}..........+\frac{1}{n^2}<2-\frac{1}{n}\\\)

 

Step 1

Prove true for n=1

LHS = 1/1^2 = 1

RHS = 2-1/1 = 1 LHS

Therefore true for n=1

 

Step 2

Assume true for n=k

 

\(1+\frac{1}{4}+\frac{1}{9}..........+\frac{1}{k^2}<2-\frac{1}{k}\\\)

 

Now prove true for n=k+1

that is, prove

\(1+\frac{1}{4}+\frac{1}{9}..........+\frac{1}{k^2}+\frac{1}{(k+1)^2}<2-\frac{1}{k+1}\\ 1+\frac{1}{4}+\frac{1}{9}..........+\frac{1}{k^2}+\frac{1}{(k+1)^2}<2-\frac{k(k+1)}{k(k+1)^2}\\ 1+\frac{1}{4}+\frac{1}{9}..........+\frac{1}{k^2}+\frac{1}{(k+1)^2}<2-\frac{k^2+1}{k(k+1)^2}\\ \\ LHS<(2-\frac{1}{k})+\frac{1}{(k+1)^2}\\ LHS<2-\left(\frac{1}{k}-\frac{1}{(k+1)^2}\right)\\ LHS<2-\left(\frac{(k+1)^2}{k(k+1)^2}-\frac{k}{k(k+1)^2}\right)\\ LHS<2-\left(\frac{k^2+k+1}{k(k+1)^2}\right)\\ LHS<2-\frac{k^2+1}{k(k+1)^2}-\frac{k}{k(k+1)^2}\\ therefore\\ LHS<2-\frac{k^2+1}{k(k+1)^2}\\ LHS<2-\frac{1}{k+1}\\ \)

Therefore, if true for n=k then it must be true for n=k+1

 

Step 3

Since true for n=1, it must be true for n=2

Since true for n=2 it must be true for n=3, n=4,n=5, etc.  

Therefore true for al n   where   \(n\in N\)

 Oct 22, 2015
 #1
avatar+21 
0

anyone?

 Oct 22, 2015
 #2
avatar+118654 
+10
Best Answer

 

Hello once again Jake   laugh

 

Prove by induction     (n is  a nattural number)

 

\(1+\frac{1}{4}+\frac{1}{9}..........+\frac{1}{n^2}<2-\frac{1}{n}\\\)

 

Step 1

Prove true for n=1

LHS = 1/1^2 = 1

RHS = 2-1/1 = 1 LHS

Therefore true for n=1

 

Step 2

Assume true for n=k

 

\(1+\frac{1}{4}+\frac{1}{9}..........+\frac{1}{k^2}<2-\frac{1}{k}\\\)

 

Now prove true for n=k+1

that is, prove

\(1+\frac{1}{4}+\frac{1}{9}..........+\frac{1}{k^2}+\frac{1}{(k+1)^2}<2-\frac{1}{k+1}\\ 1+\frac{1}{4}+\frac{1}{9}..........+\frac{1}{k^2}+\frac{1}{(k+1)^2}<2-\frac{k(k+1)}{k(k+1)^2}\\ 1+\frac{1}{4}+\frac{1}{9}..........+\frac{1}{k^2}+\frac{1}{(k+1)^2}<2-\frac{k^2+1}{k(k+1)^2}\\ \\ LHS<(2-\frac{1}{k})+\frac{1}{(k+1)^2}\\ LHS<2-\left(\frac{1}{k}-\frac{1}{(k+1)^2}\right)\\ LHS<2-\left(\frac{(k+1)^2}{k(k+1)^2}-\frac{k}{k(k+1)^2}\right)\\ LHS<2-\left(\frac{k^2+k+1}{k(k+1)^2}\right)\\ LHS<2-\frac{k^2+1}{k(k+1)^2}-\frac{k}{k(k+1)^2}\\ therefore\\ LHS<2-\frac{k^2+1}{k(k+1)^2}\\ LHS<2-\frac{1}{k+1}\\ \)

Therefore, if true for n=k then it must be true for n=k+1

 

Step 3

Since true for n=1, it must be true for n=2

Since true for n=2 it must be true for n=3, n=4,n=5, etc.  

Therefore true for al n   where   \(n\in N\)

Melody Oct 22, 2015
 #3
avatar+21 
0

wow that must've took a while thanks so much! i needed this for my homework my teacher's not a forgiving person.

 Oct 22, 2015
 #4
avatar+118654 
0

Yes well, I hope you learn from it!

I want to help you learn, it was not my intention just to do your homework for you!!

 Oct 22, 2015

4 Online Users

avatar
avatar